ChaseDream
标题: SDCAR2010【逻辑入门】(八)Assumptions [打印本页]
作者: sdcar2010 时间: 2011-6-24 02:47
标题: SDCAR2010【逻辑入门】(八)Assumptions
An assumption is an unstated premise. In other words, an assumption is a piece of evidence that strengthens the author’s argument but that the author never sates explicitly in the stimulus.
Sufficient vs. Necessary Assumptions
Imagine that you were accused of cheating during a GMAT test. At the hearing before a panel of disciplinary committee, only one piece of evidence is presented – your one-sentence testimony: “I did not cheat at the GMAT exam.” After hearing your testimony, the panel concludes that you did not cheat. You are definitely elated. But logically, the panel’s conclusion is on shaky ground because they assume, for better or worse, that you are telling the truth! In fact there are at least three different assumptions the panel could be making.
1. You always tell the truth.
2. You told the truth at the hearing.
3. It is possible for you to tell the truth.
The first assumption that you always tell the truth is a sufficient assumption because it is sufficient or enough to justify the panel’s conclusion. If you always tell the truth, then the panel must be right. However, this assumption is not a necessary assumption or an assumption that we have to make, because the panel could still be right even if you do not always tell the truth. As long as you told the truth at your hearing, the judge is still right. In other words, the panel’s conclusion does not depend on the assumption that you always tell the truth.
The second assumption that you told the truth at your hearing is both a necessary and sufficient assumption. It is sufficient because if you told the truth at the hearing, then the panel is right; it is also necessary because if you did not tell the truth at the hearing, then the panel is wrong. In other words, the panel’s decision depends on this assumption being true.
The third assumption that it is possible for you to tell the truth is a necessary assumption because if it were not possible for you to tell the truth, then the panel would be wrong. However, this assumption is not sufficient because it alone does not justify the panel’s conclusion. Just because you can tell the truth does not mean you actually tell the truth at the hearing. Therefore, the panel could still be wrong, even if the last assumption is true.
As this example illustrates, sufficient assumptions typically have strong language (always, never, none, all), while necessary assumptions typically have weak language (possible, likely, often, many).
Sufficient Assumption
Prompts for sufficient assumptions
• Which one of the following, if assumed, would justify the conclusion?
• The author’s conclusion would be properly drawn if which one of the following was true?
Sufficient assumption prompts almost always use the word IF, while necessary assumption prompts never use IF. Also, sufficient assumption prompts avoid qualifiers such as MOST. MOST JUSTIFIES, for example, is a strengthen question, while JUSTIFY by itself is a sufficient assumption question.
Before you look at the answers:
1. Pinpoint the main conclusion in the passage. (Read my previous Main Point post.)
2. Separate the premises from everything else. After you find the main point, don’t assume that all the other statements are premises; they might include opposing viewpoints, background information, and concessions.
3. Are there any jumps between the premises and the conclusion or between each premise? To find these jumps, look for any concepts that are discussed only once. There are usually two. The correct answer usually fills in the gap in the argument by linking those two concepts together. Alternatively, slowly read each word of the conclusion. Which phrase was not used in the premise? That word or phrase will likely reveal where the argument jumped to the conclusion: at the gap!
Look for the answer that links those two concepts together.
1. The correct answer will almost always use the exact same language used in the passage to describe each concept. If “many smart CDers” were one the concepts you found in the gap, the correct answer would likely use those three exact words, while the wrong answers would likely use similar but different words such as “many diligent CDers” or “smart CDers who read SCAR’s posts.” For the former, diligent is different from smart; for the latter, it refers to a smaller subset of CDers than the entire groupof “smart CDers.”
2. Because a sufficient assumption must justify the conclusion, the correct answer often has all, only, no, none, most, always, or some other strong words.
Jumps in Reasoning
Jumps can occur between the premises and the conclusion or between each premise. Here is a jump between premises:
1. Our school requires some of the highest tuitions in the country.
2. High tuitions in any school discourage poor students from attending.
3. Discriminatory schools cannot attract the best and the brightest students.
Therefore, our school cannot attract the best and the brightest students.
The first and second premises are related, but not perfectly: schools that impose the highest tuitions do not necessarily impose high tuitions – only tuitions that are higher than other schools. That said, it is not unreasonable to assume that a school with some of the highest tuitions in the country would have high tuitions. So this is a jump, but it is a small one. The second and third premises make a larger jump: a school discourages poor students from attending is not necessarily a discriminatory school.
So in a sufficient assumption question, the correct answer for this argument would likely link these last two phrases together by saying that “any school that discourages poor students from attending is a discriminatory school” or “only discriminatory schools discourage poor students from attending.” Note that both expressions say the same thing because any means if and only means then.
Necessary Assumption
Prompts for necessary assumptions:
• The conclusion relies on which one of the following assumptions?
• The argument presupposes which one of the following?
• The conclusion does not follow unless
Before you look at the answers:
1. Pinpoint the main conclusion in the passage. (Read my previous Main Point post.)
2. Separate the premises from everything else. After you find the main point, don’t assume that all the other statements are premises; they might include opposing viewpoints, background information, and concessions.
3. Are there any jumps between the premises and the conclusion or between each premise? To find these jumps, look for any concepts that are discussed only once. There often two such concepts in the same passage. Unlike the jumps in sufficient assumption passages, however, the jumps in necessary assumption passages are often less obvious.
Look for the answer that must be true for the conclusion to stand.
1. Ask yourself, “Does this answer have to be true?” Or could the conclusion still be true without it? In other words, if the answer were not true, would it undermine the conclusion? Is it an assumption that the conclusion depends on?
2. Negate your last two best answers choices. The negated answer that undermines the conclusion is the correct answer.
3. If you spot any jumps, the correct answer will often link those two concepts together.
Negating an answer choice
Imagine you are eating ice cream and your friend asks you how it tastes. You reply, “It is sweet.” The negation of this statement is that the ice cream “is not sweet.” The complete opposite of this statement is that the ice cream “is sour.”
When you negate an answer, just negate it. Try not to turn it into its opposite. Also, negate either the verb or the quantity, but not both.
SDCAR2010【逻辑入门】(七) Strengthen and Paradox
SDCAR2010【逻辑入门】(九)Flaw (part 1)
作者: jaze 时间: 2011-6-24 08:53
追
作者: horsecc 时间: 2011-6-24 09:46
good
作者: jaze 时间: 2011-6-24 09:59
1. Our school requires some of the highest tuitions in the country.
2. High tuitions in any school discourage poor students from attending.
3. Discriminatory schools cannot attract the best and the brightest students.
1. E -> F
2. A -> B
3. C -> D
conslusion: E --> D
first gap: F --> A
second gap: B --> C
but at the first glance, i thought B = D, and then i got E--> F--> A--> B = D.
Is it right as another direction of solving those questions?
作者: jaze 时间: 2011-6-24 10:14
“any school that discourages poor students from attending is a discriminatory school” or “only discriminatory schools discourage poor students from attending.” Note that both expressions say the same thing because any means if and only means then.
i make sense such things as this:
If a school that discourages poor students from attending is a discriminatory school,
then a discriminatory school discourages poor students from attending.
I think there is a assumption in this reasoning: whether a school is a discriminatory school can only defined by
whether it discourages poor students.
That is, if there is anther way for us to define a school a discriminatory school, for example, discriminate women(it is just a example, do not be serious), then, i think we can not conclude that a discriminatory school discourages poor students from attending just from the evidence that a school that discourages poor students from attending is a discriminatory school.
Is that right, if not, please correct my fault.
I just want to learn from u.
作者: sdcar2010 时间: 2011-6-24 11:29
1. Our school requires some of the highest tuitions in the country.
2. High tuitions in any school discourage poor students from attending.
3. Discriminatory schools cannot attract the best and the brightest students.
1. E -> F
2. A -> B
3. C -> D
conslusion: E --> D
first gap: F --> A
second gap: B --> C
but at the first glance, i thought B = D, and then i got E--> F--> A--> B = D.
Is it right as another direction of solving those questions?
-- by 会员 jaze (2011/6/24 9:59:14)
From the formal logic standpoint, if B-->D, then E-->D is correct. So B-->D could be a sufficient assumption.
作者: sdcar2010 时间: 2011-6-24 11:34
“any school that discourages poor students from attending is a discriminatory school” or “only discriminatory schools discourage poor students from attending.” Note that both expressions say the same thing because any means if and only means then.
What I meant is that
“any school that discourages poor students from attending is a discriminatory school” = IF a school discourages poor students from attending, then it is a discriminatory school.
"only discriminatory schools discourage poor students from attending" = THEN they are discriminatory schools if the school discourage poos students from attending.
All two statements tell the same thing.
作者: jaze 时间: 2011-6-24 21:04
Thanks for replying. I learn a lot
作者: jameshzd 时间: 2011-6-26 16:51
顶!!!
作者: jameshzd 时间: 2011-6-26 18:35
这个帖子是精华!
作者: sdcar2010 时间: 2011-6-26 21:16
Again, time to prick your brain! Enjoy. The answers and explanations will follow.
1. A recent report determined that although only 3 percent of drivers on Maryland highways equipped their vehicles with radar detectors, 33 percent of all vehicles ticketed for exceeding the speed limit were equipped with them. Clearly, drivers who equip their vehicles with radar detectors are more likely to exceed the speed limit regularly than are drivers who do not.
The conclusion drawn above depends on which of the following assumptions?
(A) Drivers who equip their vehicles with radar detectors are less likely to be ticketed for exceeding the speed limit than are drivers who do not.
(B) Drivers who are ticketed for exceeding the speed limit are more likely to exceed the speed limit regularly than are drivers who are not ticketed.
(C) The number of vehicles that were ticketed for exceeding the speed limit was greater than the number of vehicles that were equipped with radar detectors.
(D) Many of the vehicles that were ticketed for exceeding the speed limit were ticketed more than once in the time period covered by the report.
(E) Drivers on Maryland highways exceeded the speed limit more often than did drivers on other state highways not covered in the report.
2. A researcher discovered that people who have low levels of immune-system activity tend to score much lower on tests of mental health than do people with normal or high immune-system activity. The researcher concluded from this experiment that the immune system protects against mental illness as well as against physical disease.
The researcher's conclusion depends on which of the following assumptions?
(A) High immune-system activity protects against mental illness better than normal immune system activity does.
(B) Mental illness is similar to physical disease in its effects on body systems.
(C) People with high immune-system activity cannot develop mental illness.
(D) Mental illness does not cause people's immune system activity to decrease.
(E) Psychological treatment of mental illness is not as effective as is medical treatment.
3. In Kravonia, the average salary for jobs requiring a college degree has always been higher than the average salary for jobs that do not require a degree. Current enrollments in Kravonia's colleges indicate that over the next four years the percentage of the Kravonian workforce with college degrees will increase dramatically. Therefore, the average salary for all workers in Kravonia is likely to increase over the next four years.
Which of the following is an assumption on which the argument depends?
A. Kravonians with more than one college degree earn more, on average, than do Kravonians with only one college degree.
B. The percentage of Kravonians who attend college in order to earn higher salaries is higher now than it was several years ago.
C. The higher average salary for jobs requiring a college degree is not due largely to a scarcity among the Kravonian workforce of people with a college degree.
D. The average salary in Kravonia for jobs that do not require a college degree will not increase over the next four years.
E. Few members of the Kravonian workforce earned their degrees in other countries.
4. A significant number of complex repair jobs carried out by Ace Repairs have to be reworked under the company’s warranty. The reworked jobs are invariably satisfactory. When initial repairs are inadequate, therefore, it is not because the mechanics lack competence; rather, there is clearly a level of focused concentration that complex repairs require that is elicited more reliably by rework jobs than by first-time jobs.
The argument above assumes which of the following?
A. There is no systematic difference in membership between the group of mechanics who do first-time jobs and the group of those who do rework jobs.
B. There is no company that successfully competes with Ace Repairs for complex repair jobs.
C. Ace Repairs’ warranty is good on first-time jobs but does not cover rework jobs.
D. Ace Repairs does not in any way penalize mechanics who have worked on complex repair jobs that later had to be reworked.
E. There is no category of repair jobs in which Ace Repairs invariably carries out first-time jobs satisfactorily.
5. Excavations of the Roman city of Sepphoris have uncovered numerous detailed mosaics depicting several readily identifiable animal species: a hare, a partridge, and various Mediterranean fish. Oddly, most of the species represented did not live in the Sepphoris region when these mosaics were created. Since identical motifs appear in mosaics found in other Roman cities, however, the mosaics of Sepphoris were very likely created by traveling artisans from some other part of the Roman Empire.
Which of the following is an assumption on which the argument depends?
A. The Sepphoris mosaics are not composed exclusively of types of stones found naturally in the Sepphoris area.
B. There is no single region to which all the species depicted in the Sepphoris mosaics are native.
C. No motifs appear in the Sepphoris mosaics that do not also appear in the mosaics of some other Roman city.
D. All of the animal figures in the Sepphoris mosaics are readily identifiable as representations of known species.
E. There was not a common repertory of mosaic designs with which artisans who lived in various parts of the Roman Empire were familiar.
6. Newspaper editorial:
In an attempt to reduce the crime rate, the governor is getting tough on criminals and making prison conditions harsher. Part of this effort has been to deny inmates the access they formerly had to college-level courses. However, this action is clearly counter to the governor’s ultimate goal, since after being released form prison, inmates who had taken such courses committed far fewer crimes overall than other inmates.
Which of the following is an assumption on which the argument depends?
A. Not being able to take college-level courses while in prison is unlikely to deter anyone from a crime that he or she might otherwise have committed.
B. Former inmates are no more likely to commit crimes than are members of the general population.
C. The group of inmates who chose to take college-level courses were not already less likely than other inmates to commit crimes after being released.
D. Taking high school level courses in prison has less effect on an inmate’s subsequent behavior than taking college-level courses does.
E. The governor’s ultimate goal actually is to gain popularity by convincing people that something effective is being done about crime.
7. Radio stations with radio data system (RDS) technology broadcast special program information that only radios with an RDS feature can receive. Between 1994 and 1996, the number of RDS radio stations in Verdland increased from 250 to 600.However, since the number of RDS-equipped radios in Verdland was about the same in 1996 as in 1994, the number of Verdlanders receiving the special program information probably did not increase significantly.
Which of the following is an assumption on which the argument depends?
(A) Few if any of the RDS radio stations that began broadcasting in Verdland after 1994 broadcast to people with RDS-equipped radios living in areas not previously reached by RDS stations.
(B) In 1996 most Verdlanders who lived within the listening area of an RDS station already had a radio equipped to receive RDS.
(C) Equipping a radio station with RDS technology does not decrease the station's listening area.
(D) In 1996 Verdlanders who did not own radios equipped to receive RDS could not receive any programming from the RDS radio stations that began broadcasting in Verdland after 1994.
(E) The RDS radio stations in Verdland in 1996 did not all offer the same type of programming.
8. When people evade income taxes by not declaring taxable income, a vicious cycle results. Tax evasion forces lawmakers to raise income tax rates, which causes the tax burden on non-evading taxpayers to become heavier. This, in turn, encourages even more taxpayers to evade income taxes by hiding taxable income.
The vicious cycle described above could not result unless which of the following were true?
(A) An increase in tax rates tends to function as an incentive for taxpayers to try to increase their pretax incomes.
(B) Some methods for detecting tax evaders, and thus recovering some tax revenue lost through evasion, bring in more than they cost, but their success rate varies from year to year.
(C) When lawmakers establish income tax rates in order to generate a certain level of revenue, they do not allow adequately for revenue that will be lost through evasion.
(D) No one who routinely hides some taxable income can be induced by a lowering of tax rates to stop hiding such income unless fines for evaders are raised at the same time.
(E) Taxpayers do not differ from each other with respect to the rate of taxation that will cause them to evade taxes.
9. The average hourly wage of television assemblers in Vernland has long been significantly lower than that in neighboring Borodia. Since Borodia dropped all tariffs on Vernlandian televisions three years ago, the number of televisions sold annually in Borodia has not changed. However, recent statistics show a drop in the number of television assemblers in Borodia. Therefore, updated trade statistics will probably indicate that the number of televisions Borodia imports annually from Vernland has increased.
Which of the following is an assumption on which the argument depends?
(A) The number of television assemblers in Vernland has increased by at least as much as the number of television assemblers in Borodia has decreased.
(B) Televisions assembled in Vernland have features that televisions assembled in Borodia do not have.
(C) The average number of hours it takes a Borodian television assembler to assemble a television has not decreased significantly during the past three years.
(D) The number of televisions assembled annually in Vernland has increased signifi cantly during the past three years.
(E) The difference between the hourly wage of television assemblers in Vernland and the hourly wage of television assemblers in Borodia is likely to decrease in the next few years.
10. Many people suffer an allergic reaction to certain sulfites, including those that are commonly added to wine as preservatives. However, since there are several wine makers who add sulfites to none of the wines they produce, people who would like to drink wine but are allergic to sulfites can drink wines produced by these wine makers without risking an allergic reaction to sulfites.
Which of the following is an assumption on which the argument depends?
A. These wine makers have been able to duplicate the preservative effect produced by adding sulfites by means that do not involve adding any potentially allergenic substances to their wine.
B. Not all forms of sulfite are equally likely to produce the allergic reactions.
C. Wine is the only beverage to which sulfites are commonly added.
D. Apart from sulfites, there are no substances commonly present in wine that give rise to an allergic reaction.
E. Sulfites are not naturally present in the wines produced by these wine makers in amounts large enough to produce an allergic reaction in someone who drinks these wines.
作者: jameshzd 时间: 2011-6-27 09:58
my answers : B B C A C C D C C E
Waiting for the correct answers ~
作者: sdcar2010 时间: 2011-6-27 10:03
my answers : B B C A C C D C C E
Waiting for the correct answers ~
-- by 会员 jameshzd (2011/6/27 9:58:06)
#2, #5 and #7 need another look.
作者: sdcar2010 时间: 2011-6-27 10:04
1. A recent report determined that although only 3 percent of drivers on Maryland highways equipped their vehicles with radar detectors, 33 percent of all vehicles ticketed for exceeding the speed limit were equipped with them. Clearly, drivers who equip their vehicles with radar detectors are more likely to exceed the speed limit regularly than are drivers who do not.
The conclusion drawn above depends on which of the following assumptions?
(A) Drivers who equip their vehicles with radar detectors are less likely to be ticketed for exceeding the speed limit than are drivers who do not.
(B) Drivers who are ticketed for exceeding the speed limit are more likely to exceed the speed limit regularly than are drivers who are not ticketed.
(C) The number of vehicles that were ticketed for exceeding the speed limit was greater than the number of vehicles that were equipped with radar detectors.
(D) Many of the vehicles that were ticketed for exceeding the speed limit were ticketed more than once in the time period covered by the report.
(E) Drivers on Maryland highways exceeded the speed limit more often than did drivers on other state highways not covered in the report.
If you negate (B), you have:
Drivers who are ticketed for exceeding the speed limit are NOT more likely to exceed the speed limit regularly than are drivers who are not ticketed.
If that is the case, then for those drivers who have radar detectors and who got a speeding ticket, they are NOT more likely to exceed the speed limit regularly than are drivers who are not ticketed. This is in contrary to the conclusion of the stimulus -- Drivers who equip their vehicles with radar detectors ARE more likely to exceed the speed limit regularly than are driver who do not. So if you negate (B), the stimulus falls apart. Thus, (B) is the correct answer.
2. A researcher discovered that people who have low levels of immune-system activity tend to score much lower on tests of mental health than do people with normal or high immune-system activity. The researcher concluded from this experiment that the immune system protects against mental illness as well as against physical disease.
The researcher's conclusion depends on which of the following assumptions?
(A) High immune-system activity protects against mental illness better than normal immune system activity does.
(B) Mental illness is similar to physical disease in its effects on body systems.
(C) People with high immune-system activity cannot develop mental illness.
(D) Mental illness does not cause people's immune system activity to decrease.
(E) Psychological treatment of mental illness is not as effective as is medical treatment.
If you negate (D), you have:
Mental illness DOES cause people's immune system activity to decrease.
If that is true, then the author’s conclusion that the immune system protects against mental illness is wrong. So if you negate (D), the stimulus falls apart. Thus, (D) is the correct answer.
3. In Kravonia, the average salary for jobs requiring a college degree has always been higher than the average salary for jobs that do not require a degree. Current enrollments in Kravonia's colleges indicate that over the next four years the percentage of the Kravonian workforce with college degrees will increase dramatically. Therefore, the average salary for all workers in Kravonia is likely to increase over the next four years.
Which of the following is an assumption on which the argument depends?
A. Kravonians with more than one college degree earn more, on average, than do Kravonians with only one college degree.
B. The percentage of Kravonians who attend college in order to earn higher salaries is higher now than it was several years ago.
C. The higher average salary for jobs requiring a college degree is not due largely to a scarcity among the Kravonian workforce of people with a college degree.
D. The average salary in Kravonia for jobs that do not require a college degree will not increase over the next four years.
E. Few members of the Kravonian workforce earned their degrees in other countries.
If you negate C), you get:
The higher average salary for jobs requiring a college degree is due largely to a scarcity among the Kravonian workforce of people with a college degree.
If this is the case, with the dramatically increasing percentage of Kravonian workforce with college degrees, the shortage of college graduates will be history, then so will be the higher average salary for jobs requiring a college degree. If this is the case, the conclusion that the average salary for all workers is likely to increase will not hold.
4. A significant number of complex repair jobs carried out by Ace Repairs have to be reworked under the company’s warranty. The reworked jobs are invariably satisfactory. When initial repairs are inadequate, therefore, it is not because the mechanics lack competence; rather, there is clearly a level of focused concentration that complex repairs require that is elicited more reliably by rework jobs than by first-time jobs.
The argument above assumes which of the following?
A. There is no systematic difference in membership between the group of mechanics who do first-time jobs and the group of those who do rework jobs.
B. There is no company that successfully competes with Ace Repairs for complex repair jobs.
C. Ace Repairs’ warranty is good on first-time jobs but does not cover rework jobs.
D. Ace Repairs does not in any way penalize mechanics who have worked on complex repair jobs that later had to be reworked.
E. There is no category of repair jobs in which Ace Repairs invariably carries out first-time jobs satisfactorily.
Part of the main conclusion says: When initial repairs are inadequate, therefore, it is not because the mechanics lack competence.
If you negate A), you have:
There is SOME systematic difference in membership between the group of mechanics who do first-time jobs and the group of those who do rework jobs.
If this is true, there is a difference in skills between the two groups of mechanics and the first group might be less competent than the second and the reason why the first repair fails is due to incompetence. Thus, the conclusion of the argument falls apart.
5. Excavations of the Roman city of Sepphoris have uncovered numerous detailed mosaics depicting several readily identifiable animal species: a hare, a partridge, and various Mediterranean fish. Oddly, most of the species represented did not live in the Sepphoris region when these mosaics were created. Since identical motifs appear in mosaics found in other Roman cities, however, the mosaics of Sepphoris were very likely created by traveling artisans from some other part of the Roman Empire.
Which of the following is an assumption on which the argument depends?
A. The Sepphoris mosaics are not composed exclusively of types of stones found naturally in the Sepphoris area.
B. There is no single region to which all the species depicted in the Sepphoris mosaics are native.
C. No motifs appear in the Sepphoris mosaics that do not also appear in the mosaics of some other Roman city.
D. All of the animal figures in the Sepphoris mosaics are readily identifiable as representations of known species.
E. There was not a common repertory of mosaic designs with which artisans who lived in various parts of the Roman Empire were familiar.
If you negate E), you have:
There was a common repertory of mosaic designs with which artisans who lived in various parts of the Roman Empire were familiar
If so, the artists do not need to travel and would still be able to create the mosaic designs in Seppphoris. Thus the conclusion falls apart.
6. Newspaper editorial:
In an attempt to reduce the crime rate, the governor is getting tough on criminals and making prison conditions harsher. Part of this effort has been to deny inmates the access they formerly had to college-level courses. However, this action is clearly counter to the governor’s ultimate goal, since after being released form prison, inmates who had taken such courses committed far fewer crimes overall than other inmates.
Which of the following is an assumption on which the argument depends?
A. Not being able to take college-level courses while in prison is unlikely to deter anyone from a crime that he or she might otherwise have committed.
B. Former inmates are no more likely to commit crimes than are members of the general population.
C. The group of inmates who chose to take college-level courses were not already less likely than other inmates to commit crimes after being released.
D. Taking high school level courses in prison has less effect on an inmate’s subsequent behavior than taking college-level courses does.
E. The governor’s ultimate goal actually is to gain popularity by convincing people that something effective is being done about crime.
If you negate C), you get: The group of inmates who chose to take college-level courses WERE already less likely than other inmates to commit crimes after being released. If so, these inmates are GOOD inmates to begin with, with or without the college-level course training. Therefore, the argument that allowing an inmate to take such courses in prison would decrease an inmate's chance of committing crimes after his or her release is false.
Thus, C) is the necessary assumption.
7. Radio stations with radio data system (RDS) technology broadcast special program information that only radios with an RDS feature can receive. Between 1994 and 1996, the number of RDS radio stations in Verdland increased from 250 to 600.However, since the number of RDS-equipped radios in Verdland was about the same in 1996 as in 1994, the number of Verdlanders receiving the special program information probably did not increase significantly.
Which of the following is an assumption on which the argument depends?
(A) Few if any of the RDS radio stations that began broadcasting in Verdland after 1994 broadcast to people with RDS-equipped radios living in areas not previously reached by RDS stations.
(B) In 1996 most Verdlanders who lived within the listening area of an RDS station already had a radio equipped to receive RDS.
(C) Equipping a radio station with RDS technology does not decrease the station's listening area.
(D) In 1996 Verdlanders who did not own radios equipped to receive RDS could not receive any programming from the RDS radio stations that began broadcasting in Verdland after 1994.
(E) The RDS radio stations in Verdland in 1996 did not all offer the same type of programming.
If you negate A), you have:
Few if any of the RDS radio stations that began broadcasting in Verdland after 1994 DID NOT broadcast to people with RDS-equipped radios living in areas not previously reached by RDS stations.
This is a double negative statement. If that is true, then almost all these stations broadcast to people with RDS-equipped radios living in areas not previously reached by RDS stations. In other words, stations reached out to NEW listeners! If so, the conclusion “the number of Verdlanders receiving the special program information probably did not increase significantly” is wrong. Thus, A) is the necessary assumption.
8. When people evade income taxes by not declaring taxable income, a vicious cycle results. Tax evasion forces lawmakers to raise income tax rates, which causes the tax burden on non-evading taxpayers to become heavier. This, in turn, encourages even more taxpayers to evade income taxes by hiding taxable income.
The vicious cycle described above could not result unless which of the following were true?
(A) An increase in tax rates tends to function as an incentive for taxpayers to try to increase their pretax incomes.
(B) Some methods for detecting tax evaders, and thus recovering some tax revenue lost through evasion, bring in more than they cost, but their success rate varies from year to year.
(C) When lawmakers establish income tax rates in order to generate a certain level of revenue, they do not allow adequately for revenue that will be lost through evasion.
(D) No one who routinely hides some taxable income can be induced by a lowering of tax rates to stop hiding such income unless fines for evaders are raised at the same time.
(E) Taxpayers do not differ from each other with respect to the rate of taxation that will cause them to evade taxes.
If you negate C), you have:
When lawmakers establish income tax rates in order to generate a certain level of revenue, they DO allow adequately for revenue that will be lost through evasion.
If this is true, the vicious cycle of – evading taxes --> tax shortfall --> raising taxes --> more people evading taxes --would not happen, because the trigger of the cycle – evading taxes would lead to tax shortfall – would not occur. Thus, the whole argument falls apart.
9. The average hourly wage of television assemblers in Vernland has long been significantly lower than that in neighboring Borodia. Since Borodia dropped all tariffs on Vernlandian televisions three years ago, the number of televisions sold annually in Borodia has not changed. However, recent statistics show a drop in the number of television assemblers in Borodia. Therefore, updated trade statistics will probably indicate that the number of televisions Borodia imports annually from Vernland has increased.
Which of the following is an assumption on which the argument depends?
(A) The number of television assemblers in Vernland has increased by at least as much as the number of television assemblers in Borodia has decreased.
(B) Televisions assembled in Vernland have features that televisions assembled in Borodia do not have.
(C) The average number of hours it takes a Borodian television assembler to assemble a television has not decreased significantly during the past three years.
(D) The number of televisions assembled annually in Vernland has increased signifi cantly during the past three years.
(E) The difference between the hourly wage of television assemblers in Vernland and the hourly wage of television assemblers in Borodia is likely to decrease in the next few years.
Premise: Televisions are made in both V and B, and B imports TV from V. The number of televisions sold in B keeps constant. These TV's come from both B and V. The number of TV assemblers in B decreases.
Conclusion: TV imports from V to B have increased.
The assumption is that if TV assemblers in B are fewer, there will be fewer TV made in B. A further assumption is that the efficiency of TV assemblers in B has not changed (answer C).
If you use negation method, you would find that after negating C), the argument falls apart. If assemblers in B become more efficient, they can produce the same amount or more TV than before, even with a lower number of total assemblers.
10. Many people suffer an allergic reaction to certain sulfites, including those that are commonly added to wine as preservatives. However, since there are several wine makers who add sulfites to none of the wines they produce, people who would like to drink wine but are allergic to sulfites can drink wines produced by these wine makers without risking an allergic reaction to sulfites.
Which of the following is an assumption on which the argument depends?
A. These wine makers have been able to duplicate the preservative effect produced by adding sulfites by means that do not involve adding any potentially allergenic substances to their wine.
B. Not all forms of sulfite are equally likely to produce the allergic reactions.
C. Wine is the only beverage to which sulfites are commonly added.
D. Apart from sulfites, there are no substances commonly present in wine that give rise to an allergic reaction.
E. Sulfites are not naturally present in the wines produced by these wine makers in amounts large enough to produce an allergic reaction in someone who drinks these wines.
If you negate E), you have:
sulfites ARE naturally present in the wines in large enough amount to cause allergic reaction.
If this is true, those wine without added preservatives are also problematic because the NATURALLY occurring sulfites. Then the argument that allergic people can avoid sulfites by choosing wines without preservatives will not hold. Hence, E) is a necessary assumption.
作者: jaze 时间: 2011-6-27 10:48
Difficult those questions are . Although i know all answers to the questions, i know i am not able to get the right answers when in test. Because i just know the answer, i do not know how to get the right answer and how to solve some types question if all the stuffs in the question are changed. That is, i do not understand the principle of solving those questions.
For example
question 7
I know the answer is A, but i do not think i can read out the answer in the test day. I even can not figure out what it says unless i read it over and over again. Time is wasted like that.
How can i read out the meaning as like i was reading a Chinese fiction? how to get ?
作者: sdcar2010 时间: 2011-6-27 10:53
7. Radio stations with radio data system (RDS) technology broadcast special program information that only radios with an RDS feature can receive (background info). Between 1994 and 1996, the number of RDS radio stations in Verdland increased from 250 to 600 (premise 1). However, sincethe number of RDS-equipped radios in Verdland was about the same in 1996 as in 1994 (premise 2), the number of Verdlanders receiving the special program information probably did not increase significantly (main conclusion).
This passage adopts the typical argument structure: Background. Pemise. However, premise, main conclusion. Verdland and Verdlanders are just like Beijing and Beijingness. Train your eyes to spot premises and conclusion. They would help you choose the right answers.
Fore general understanding of English passages, you have to read then re-read difficult passages. No shortcut.
作者: jameshzd 时间: 2011-6-27 11:45
For question 7
ii state why i choose C: i originally thought that the number of RDS-equipped radios can not dictate the the number of listeners, if new listeners that don't own a RDS-equipped radio can share the RDS-equipped radio with one who owned it . In this way, new listeners can receive the progamming information despite the fixed numeber of RDS-equipped radios , thus refuting the conclusion.
But after closely examined the option C, i think it is wrong becasue it cunningly leaves out the word "information" which should have been added after "could not receive any programming". As the option stated, if i negate it, it becomes" In 1996 Verdlanders who did not own radios equipped to receive RDS could receive any programming" it contradicts the background the argument provided by itself; that is one who did not own radio is impossible to receive programming because its exclusivity to the RDS-equipped radio . But if c add the word "information" after "could not receive any programming". Would C be right ? hope for your suggestion !
作者: jameshzd 时间: 2011-6-27 12:04
i choose D, not C, i always make such silly mistake.hehe...
作者: sdcar2010 时间: 2011-6-27 12:14
I think D) simply repeats the first sentence of the stimulus, which is background informaion. Therefore, D) is not an assumption, which has to be an unstated premise.
作者: jameshzd 时间: 2011-6-27 12:33
Thank you Sdcar2010 ! i learn a lot !
作者: perain 时间: 2011-6-27 13:23
I think the Q1 should note that the car equips the radar detector to avoid the tickets....I wouldn't make a mistake here if the question provides this background info...
作者: sdcar2010 时间: 2011-6-27 21:30
I think the Q1 should note that the car equips the radar detector to avoid the tickets....I wouldn't make a mistake here if the question provides this background info...
-- by 会员 perain (2011/6/27 13:23:24)
GMAC treat the purpose of a radar detector as a common sense in the same way it treats that of a toothbrush.
作者: sdcar2010 时间: 2011-6-28 11:32
The higher the level of certain vitamins and minerals in the bloodstream, the better a person's lung function, as measured by the amount of air the person can expel in one second. The lung function of smokers is significantly worse, on average, than that of nonsmoker. Clearly, therefore, one way for smokers to improve their lung function is for them to increase their intake of foods that are rich in these helpful vitamins and minerals.
Which of the following is an assumption on which this argument depends?
A) Smokers are less likely than nonsmokers to have diets that are rich in vitamins and minerals
B) The lung function of smokers whose diet are rich in those vitamins and minerals is generally better than that of nonsmokers with comparable diets
C) People whose diets are deficient in those vitamins and minerals do not typically have other health problems in addition to diminished lung function.
D) Stopping smoking will not typically improve lung function more than any diet changes can.
E) Smoking does not introduce into the body chemicals that prevent the helpful vitamins and minerals from entering the bloodstream
Anyone what to try the negation method on this question and find the correct answer?
作者: perain 时间: 2011-6-28 15:49
I think the answer is E. If I make a mistake, please correct me~~
If negate E, we get:"Smoking DOES introduce into the body chemicals that prevent the helpful vitamins and minerals from entering the bloodstream". In this case, even though the smokers intake foods that are rich in vitamins and minerals, these chemicals will prevent the V/Ms from entering the bloodstream, and V/Ms become no use. Then the conclusion is impossible to achieve.
作者: sdcar2010 时间: 2011-6-28 19:58
I think the answer is E. If I make a mistake, please correct me~~
If negate E, we get:"Smoking DOES introduce into the body chemicals that prevent the helpful vitamins and minerals from entering the bloodstream". In this case, even though the smokers intake foods that are rich in vitamins and minerals, these chemicals will prevent the V/Ms from entering the bloodstream, and V/Ms become no use. Then the conclusion is impossible to achieve.
-- by 会员 perain (2011/6/28 15:49:53)
Correct! Very logical.
作者: sdcar2010 时间: 2011-6-28 20:58
Try this question:
The U.S. census is not perfect: thousands of Americans probably go uncounted. However, the basic statistical portrait of the nation painted by the census is accurate. Certainly some of the poor go uncounted, particularly the homeless; but some of the rich go uncounted as well, because they are often abroad or traveling between one residence and another.
Which of the following is an assumption on which the argument above depends?
(A) Both the rich and the poor have personal and economic reasons to avoid being counted by the census.
(B) All Americans may reasonably be classified as either poor or rich.
(C) The percentage of poor Americans uncounted by the census is close to the percentage of rich Americans uncounted.
(D) The number of homeless Americans is approximately equal to the number of rich Americans.
(E) The primary purpose of the census is to analyze the economic status of the American population.
作者: 小小照妖镜 时间: 2011-6-28 21:33
顶!
作者: perain 时间: 2011-6-29 09:56
My answer for the U.S. census problem: C
I guess the "the basic statistical portrait" means the proportion of each group to the entire population, or something close...
作者: sdcar2010 时间: 2011-6-29 10:56
My answer for the U.S. census problem: C
I guess the "
the basic statistical portrait" means the proportion of each group to the entire population, or something close...-- by 会员 perain (2011/6/29 9:56:04)
Right again. You are on a roll.
作者: 若雪 时间: 2011-6-29 11:21
Wow!! Online teaching lesson ah? Mark to join ~~
作者: sdcar2010 时间: 2011-6-30 19:01
If a question asks for NECESSARY assumption, you can try the negation method. By definition, a necessary assumtion is a claim which is guaranteed to be correct if the original argument is true (thus the modifier NECESSARY). In terms of logic:
If the orginal argument is correct, then the NECESSARY assumption is true.
The countrapositive of the above statement is:
If the NECESSARGY assumption is NOT true (negation), the orginal argument is NOT correct.
So if you negate an answer choice, the original argument falls apart, then that choice is the correct answer for a necessary assumption.
作者: guoqi 时间: 2011-7-1 06:13
标题: SDCAR2010 another question
Hey, nice to talk to you again,
As for these assumption questions, the general rule is to negate the option to see whether the option can weaken the conclustion. My question is I find lost of questions follow the principle like this:
If A( premise), Then B ( Conclusion)
What we need to do is find a necessary assumption C to prove if without C, B doesn't exit. Right?
But, what is the role of A? , " if, then" sounds like a scieficent premise, But I doubt A is a scieficent premise.... I don't know in such kind of the question, what can A do since C is a necessary assumption. Just want to know that, do you think that I have gone too far beyond the scope?
Thanks
作者: sdcar2010 时间: 2011-7-1 11:10
If A, then B.
A is a premise and B is a conclusion. A is a sufficient condition for B to occur; B is a necessary condition to happen if A happens. That's it.
作者: sanlingjie 时间: 2011-7-2 19:16
要崩溃了,十道题快错光了!!!!!!啊!!!!!要疯了!!!!
作者: sdcar2010 时间: 2011-7-2 21:26
要崩溃了,十道题快错光了!!!!!!啊!!!!!要疯了!!!!
-- by 会员 sanlingjie (2011/7/2 19:16:38)
I felt your pain. But you have to do it again.
作者: sanlingjie 时间: 2011-7-3 00:05
要崩溃了,十道题快错光了!!!!!!啊!!!!!要疯了!!!!
-- by 会员 sanlingjie (2011/7/2 19:16:38)
I felt your pain. But you have to do it again.
-- by 会员 sdcar2010 (2011/7/2 21:26:13)
刚刚看了你2010的那些逻辑题分析的帖子,结果几乎每一题我都做错了,我真快疯了,差点把自己的脑袋垂爆!我怎么这么笨啊!楼主能不能教教我该怎么办啊!我今天一天都在看你的帖子,每篇都很认真地看,可是一旦做你出的那些练习题,就全完蛋了。我太伤心了!
作者: sdcar2010 时间: 2011-7-3 03:16
No shortcut. Redo the problems and figure out why you chose the wrong answers.
作者: jianaozhonghua 时间: 2011-7-4 02:48
无法区分sufficient和necessary。谁能给我讲讲怎么区别?使用negate的时候应该注意什么。。。
作者: Vicky90 时间: 2011-7-6 19:54
Try this question:
The U.S. census is not perfect: thousands of Americans probably go uncounted. However, the basic statistical portrait of the nation painted by the census is accurate. Certainly some of the poor go uncounted, particularly the homeless; but some of the rich go uncounted as well, because they are often abroad or traveling between one residence and another.
Which of the following is an assumption on which the argument above depends?
(A) Both the rich and the poor have personal and economic reasons to avoid being counted by the census.
(B) All Americans may reasonably be classified as either poor or rich.
(C) The percentage of poor Americans uncounted by the census is close to the percentage of rich Americans uncounted.
(D) The number of homeless Americans is approximately equal to the number of rich Americans.
(E) The primary purpose of the census is to analyze the economic status of the American population.
-- by 会员 sdcar2010 (2011/6/28 20:58:35)
Which is a accurate answer ? I confused by the option of A and E .
作者: sdcar2010 时间: 2011-7-6 22:30
C) is the answer for the U.S. census question.
作者: 米样 时间: 2011-7-9 16:46
这一部分真是我的软肋啊,看这篇帖子都比较吃力。。。
作者: dualtunnal 时间: 2011-7-15 22:15
牛牛,sufficient和necessary分不清啊,牛牛给的例子看得很清楚,不过出在题目里面就分不清啦,感觉justify the conclusion 和 rely on the assumption差不多啊,求解答~
作者: glendasc 时间: 2011-7-15 23:25
For question No.2
2. A researcher discovered that people who have low levels of immune-system activity tend to score much lower on tests of mental health than do people with normal or high immune-system activity. The researcher concluded from this experiment that the immune system protects against mental illness as well as against physical disease.
The researcher's conclusion depends on which of the following assumptions?
(A) High immune-system activity protects against mental illness better than normal immune system activity does.
(B) Mental illness is similar to physical disease in its effects on body systems.
(C) People with high immune-system activity cannot develop mental illness.
(D) Mental illness does not cause people's immune system activity to decrease.
(E) Psychological treatment of mental illness is not as effective as is medical treatment.
But if you negate C. people with high immune-system activity CAN develop mental illness. If that is true, then the conclusion that the immune system protects against mental illness is wrong. So why choose D but not c?
作者: sdcar2010 时间: 2011-7-16 00:02
For question No.2
2. A researcher discovered that people who have low levels of immune-system activity tend to score much lower on tests of mental health than do people with normal or high immune-system activity. The researcher concluded from this experiment that the immune system protects against mental illness as well as against physical disease.
The researcher's conclusion depends on which of the following assumptions?
(A) High immune-system activity protects against mental illness better than normal immune system activity does.
(B) Mental illness is similar to physical disease in its effects on body systems.
(C) People with high immune-system activity cannot develop mental illness.
(D) Mental illness does not cause people's immune system activity to decrease.
(E) Psychological treatment of mental illness is not as effective as is medical treatment.
But if you negate C. people with high immune-system activity CAN develop mental illness. If that is true, then the conclusion that the immune system protects against mental illness is wrong. So why choose D but not c?
-- by 会员 glendasc (2011/7/15 23:25:01)
If you negate C), the conclusion still holds since the conclusion is "the immune system PROTECTs against mental illness,", which means people with high immune system have some protection but that protection is not bullet-proof. So people with high immune system could become mentally unstable, but less likely than those who have low immune system.
作者: sdcar2010 时间: 2011-7-16 00:03
牛牛,sufficient和necessary分不清啊,牛牛给的例子看得很清楚,不过出在题目里面就分不清啦,感觉justify the conclusion 和 rely on the assumption差不多啊,求解答~
-- by 会员 dualtunnal (2011/7/15 22:15:12)
justify the conclusion: sufficient assumption
rely on the assumption: necessary assumption
作者: srosalita 时间: 2011-7-21 13:39
For question No.2
2. A researcher discovered that people who have low levels of immune-system activity tend to score much lower on tests of mental health than do people with normal or high immune-system activity. The researcher concluded from this experiment that the immune system protects against mental illness as well as against physical disease.
The researcher's conclusion depends on which of the following assumptions?
(A) High immune-system activity protects against mental illness better than normal immune system activity does.
(B) Mental illness is similar to physical disease in its effects on body systems.
(C) People with high immune-system activity cannot develop mental illness.
(D) Mental illness does not cause people's immune system activity to decrease.
(E) Psychological treatment of mental illness is not as effective as is medical treatment.
But if you negate C. people with high immune-system activity CAN develop mental illness. If that is true, then the conclusion that the immune system protects against mental illness is wrong. So why choose D but not c?
-- by 会员 glendasc (2011/7/15 23:25:01)
If you negate C), the conclusion still holds since the conclusion is "the immune system PROTECTs against mental illness,", which means people with high immune system have some protection but that protection is not bullet-proof. So people with high immune system could become mentally unstable, but less likely than those who have low immune system.
-- by 会员 sdcar2010 (2011/7/16 0:02:26)
Dear scdcar2010,
Can you tell me how to negate this sentence "children who don't spend all of their after-school hours playing computer games spend at least some of that time talking with other people." Can i translate it into: children who don't spend all of their after-school hours playing computer games spend most of that time talking with other people?
Besides, how to use a negated sentence to undermine the conclusion, is it need to corporate the existed premises to undermine it?
作者: sdcar2010 时间: 2011-7-21 20:24
children who don't spend all of their after-school hours playing computer games spend NONE of that time talking with other people.
作者: sdcar2010 时间: 2011-7-21 20:24
children who don't spend all of their after-school hours playing computer games DO NOT spend at least some of that time talking with other people
作者: rongyang114 时间: 2011-7-21 22:49
sdcar,可以转帖吗?更多人可以学习到~
作者: AlohaDJ 时间: 2011-7-22 00:40
...
作者: AlohaDJ 时间: 2011-7-22 00:44
My answer for the U.S. census problem: C
I guess the "
the basic statistical portrait" means the proportion of each group to the entire population, or something close...-- by 会员 perain (2011/6/29 9:56:04)
Right again. You are on a roll.
-- by 会员 sdcar2010 (2011/6/29 10:56:15)
NN, could u plz give a little more light on "the basic statistical portrait"?
It means the proportion of each group to the entire population, or something close...How can we get it? another common sense??
THKS!
作者: srosalita 时间: 2011-7-22 09:39
children who don't spend all of their after-school hours playing computer games DO NOT spend at least some of that time talking with other people
-- by 会员 sdcar2010 (2011/7/21 20:24:36)
THX!
作者: sdcar2010 时间: 2011-7-22 11:45
My answer for the U.S. census problem: C
I guess the "
the basic statistical portrait" means the proportion of each group to the entire population, or something close...-- by 会员 perain (2011/6/29 9:56:04)
Right again. You are on a roll.
-- by 会员 sdcar2010 (2011/6/29 10:56:15)
NN, could u plz give a little more light on "
the basic statistical portrait"?
It means the proportion of each group to the entire population, or something close...How can we get it? another common sense??
THKS!-- by 会员 AlohaDJ (2011/7/22 0:44:22)
Good question. The basic statistical portrait might mean a statistical trend, a statistical discription, a statistical map.
作者: AlohaDJ 时间: 2011-7-23 17:47
Good question. The basic statistical portrait might mean a statistical trend, a statistical discription, a statistical map.
-- by 会员 sdcar2010 (2011/7/22 11:45:03)
THKS a lot!
作者: 三仙包 时间: 2011-7-24 12:02
太精彩啦~~~不甚感激~~!!牛牛~~爱死你了!!
作者: mia113 时间: 2011-7-27 11:36
up up up up !!!!!!!!!!!!!!!!!!
作者: mia113 时间: 2011-7-27 14:18
For question 9`
When I took the exercise, I thought C was irrelevant, so I eliminated it withoout thinking carefully.
I think the premises of question 9 are: TV sales in B don't change, TV assembled in B decrease
conclusion: B import more TV from C
negate AD can make the conclusion fall apart.
why these mistakes took place?
Could you please tell me how to tell the relevant and irrelevant options. Thanke you``
作者: feifan1984 时间: 2011-7-29 10:47
跟看美剧一样追楼主的帖子...
作者: mia113 时间: 2011-7-29 10:49
今天重新看了9题·
发现当时把
assemblers看成了assembled了··
文章的推理: assemblers in B decrease --> television imported from V increased
中间有个gap: drop in assembler -->drop in television asselbed in B which is related to the efficiency of the assemblers
仔细想想当时自己错的原因是认为 B地卖的TV不变· TV assembled in B decrease (当时的想法,后来证实是错的), tv imported from V increase
自己当时觉得有个gap: B的减少,不能推出 减少量就是来自V 心想如果可以找到能够证明 B的减少就是V增加的就可以补全这个GAP
但是后来看LZ在问答别人问题时·有一点 :"These TV's come from both B and V"
请问下楼主通过看文章 是怎么得出这个隐含条件的吖?
觉得自己有时候 分不清 怎么是反驳了premise,反驳推理链,weaken evidence
希望能够得到您的解答··谢谢了·······
作者: carrick0526 时间: 2011-8-2 23:18
謝謝樓主的貼..我從中獲益良多
想問一下樓主給的十道題是不是都是找出necessary assumption?而不是sufficient assumption?
作者: sdcar2010 时间: 2011-8-3 00:29
謝謝樓主的貼..我從中獲益良多
想問一下樓主給的十道題是不是都是找出necessary assumption?而不是sufficient assumption?
-- by 会员 carrick0526 (2011/8/2 23:18:29)
Yes.
作者: CatCandy 时间: 2011-8-5 10:39
hi sdcar2010 , could you giv us a sufficient question and its analysis? I see most examples are necessary ones.
作者: sdcar2010 时间: 2011-8-5 12:18
The average thirty-year-old Chinese will have watched more than ten thousand TV commercials in his or her lifetime. Therefore, the TV commercial has influenced Chinese way of thinking.
The conclusion above follows logically if which one of the following is assumed?
A) The way of thinking which people develop are largely determined by external influences.
B) Anything people are exposed to in great quantity will influence their way of thinking.
C) It is impossible to avoid or ignore TV commercials.
D) Some peole find TV commercials more interesting to watch than the news broadcast every night.
E) Certain form of communication to which people are subjected will affect their way of thinking.
hi sdcar2010 , could you giv us a sufficient question and its analysis? I see most examples are necessary ones.
-- by 会员 CatCandy (2011/8/5 10:39:09)
作者: AlohaDJ 时间: 2011-8-5 14:34
My answer is B. For it fills in the gap between the premis and the conclusion.
作者: CatCandy 时间: 2011-8-5 15:32
Thx! I choose B
Another problem is that when I am doing you 28 CR questions, I use almost 1 hour to finish. Accourding to the 7/4 min per question rate, I am expected to finish within 50min. Am I so slow? What should I do? I have only 10 days before the exam...
作者: sdcar2010 时间: 2011-8-5 20:49
My 28 questions are hard questions. If it only took you one hour to finish, you are one of the best.
作者: CatCandy 时间: 2011-8-6 09:18
That does relieve my pain.Thx! You are such a great teacher!
作者: CatCandy 时间: 2011-8-6 11:31
In the years since the city of London imposed strict air-pollution regulations on local industry, the number of bird species seen in and around London has increased dramatically. Similar air-pollution rules should be imposed in other major cities.
Each of the following is an assumption made in the argument above EXCEPT:
(A) In most major cities, air-pollution problems are caused almost entirely by local industry.
(B) Air-pollution regulations on industry have a significant impact on the quality of the air.
(C) The air-pollution problems of other major cities are basically similar to those once suffered by London.
(D) An increase in the number of bird species in and around a city is desirable.
(E) The increased sightings of bird species in and around London reflect an actual increase in the number of species in the area.
I am confused about this one: why A is correct? Is it because the conclusion did not mention the pollution rules should be on the industry? Why E is wrong? I do not think it is revelent...
作者: carrick0526 时间: 2011-8-6 14:50
有十道題的第五題...
樓主給的答案C是NECESSARY ASSUMPTION嗎..
我看了樓主的答案還是不明白..感覺C不是必要條件
因為就算C不對..好像總結也沒有影響
作者: briarbaby 时间: 2011-8-10 11:37
I learn a lot from sdcar2010's post. I think sufficient assumption is more or less similar to strengthen, but I'm not sure...
作者: lianlianchen 时间: 2011-8-14 02:23
LZ的头像看了眼晕, 你之前那些美男美女的头像多养眼啊, 抽空换一换吧, 请照顾一下你的FANS的视力.
作者: 纳什总冠军 时间: 2011-8-23 15:36
非常感谢SDcar的题目和解析,不过我有如下几个问题,希望您能抽空解答一下:
1.我在做逻辑题目特别是假设题目的时候常常会因为几个比较关键的词不知道是什么意思而严重影响我对这个argument的理解,导致最后不知所云,因此想请教一下,怎样能在完全不知道argument在讲什么的时候把正确的选项选择出来?
2.关于您给出的10道necessary Assumption中的Q10,我是negativea D,结果是Apart from sulfites, there are substances commonly present in wine that give rise to an allergic reaction.我个人感觉这说明了即使有些商人没有添加s,还是有些消费者会allergic。
3.Q6的A选项, Not being able to take college-level courses while in prison is unlikely to deter anyone from a crime that he or she might otherwise have committed. 这句话中文意思是什么,取非后是否能反驳conclusion?
谢谢解答!
作者: helenlife 时间: 2011-9-13 11:30
Hi SDCAR2010,
Just get a question: What's the difference between assumption and premise?
It seems critical for certain types of CR.
Look forward to your explanation.
Thanks!
作者: sdcar2010 时间: 2011-9-14 03:53
Assumption = "unstated" premise
作者: caoqin1981 时间: 2011-9-14 08:06
这个贴让我更加清楚了什么是普通的充分条件,什么是必要且充分条件,谢谢LZ的分享
作者: waiting1 时间: 2011-9-30 20:02
5. Excavations of the Roman city of Sepphoris have uncovered numerous detailed mosaics depicting several readily identifiable animal species: a hare, a partridge, and various Mediterranean fish. Oddly, most of the species represented did not live in the Sepphoris region when these mosaics were created. Since identical motifs appear in mosaics found in other Roman cities, however, the mosaics of Sepphoris were very likely created by traveling artisans from some other part of the Roman Empire.
这题牛牛为什么选C呢?我在做gwd时遇到了这题,我自己也选的c,但答案给的是E。。。牛牛能解释下e哪里不对吗?谢谢
作者: sdcar2010 时间: 2011-9-30 22:09
I was wrong. E is right. Sorry.
作者: suntree846 时间: 2011-10-3 10:56
hello LZ:
10. Many people suffer an allergic reaction to certain sulfites, including those that are commonly added to wine as preservatives. However, since there are several wine makers who add sulfites to none of the wines they produce, people who would like to drink wine but are allergic to sulfites can drink wines produced by these wine makers without risking an allergic reaction to sulfites.
Which of the following is an assumption on which the argument depends?
A. These wine makers have been able to duplicate the preservative effect produced by adding sulfites by means that do not involve adding any potentially allergenic substances to their wine.
B. Not all forms of sulfite are equally likely to produce the allergic reactions.
C. Wine is the only beverage to which sulfites are commonly added.
D. Apart from sulfites, there are no substances commonly present in wine that give rise to an allergic reaction.
E. Sulfites are not naturally present in the wines produced by these wine makers in amounts large enough to produce an allergic reaction in someone who drinks these wines.
i can't understand what's wrong with the answer(D)
look forward to you explanation!
thx!
作者: sdcar2010 时间: 2011-10-3 12:17
Remember: focus on the conclusion, which deals with allergy to SULFITES. We do not care if there are other allergic substances in wine.
hello LZ:
10. Many people suffer an allergic reaction to certain sulfites, including those that are commonly added to wine as preservatives. However, since there are several wine makers who add sulfites to none of the wines they produce, people who would like to drink wine but are allergic to sulfites can drink wines produced by these wine makers without risking an allergic reaction to sulfites.
Which of the following is an assumption on which the argument depends?
A. These wine makers have been able to duplicate the preservative effect produced by adding sulfites by means that do not involve adding any potentially allergenic substances to their wine.
B. Not all forms of sulfite are equally likely to produce the allergic reactions.
C. Wine is the only beverage to which sulfites are commonly added.
D. Apart from sulfites, there are no substances commonly present in wine that give rise to an allergic reaction.
E. Sulfites are not naturally present in the wines produced by these wine makers in amounts large enough to produce an allergic reaction in someone who drinks these wines.
i can't understand what's wrong with the answer(D)
look forward to you explanation!
thx!
-- by 会员 suntree846 (2011/10/3 10:56:21)
作者: suntree846 时间: 2011-10-3 14:35
I see
thank you!
作者: suntree846 时间: 2011-10-3 19:30
A question again
for Q1,my idea is: a jump exits between ticketed for exceeding the speed limit and more likely to exceed the speed limit ,while the (b) links the former concept with the latter one, as a result ,the (b) is more likely the corret answer than the rest.
is that right?
Besides, after reading your series, I can got correct answers more frequently, however , I have to spend a lot of time gaining them for I have to read and understand the stimulus and the options carefully. So what should I do for my pace now? How can I practise for both the high rate of correction and ideal pace? What is the ideal speed of one CR question for the test?
Thanks
作者: sdcar2010 时间: 2011-10-3 22:08
Any gap or jump in the logic chain can be filled either with a sufficient assumption or a necessary one. So finding a gap definitely helps answer an assumption question. In that sense, your approach is a good one.
First, read the passage and find the premises and the conclusion. Then read all answer choices. Once locating all POSSIBLLY correct answer choices after eliminating those unlikely ones, your next task is to find the best among all the remaining POSSIBLE choices. This would be my approach to CR questions.
As to the pace, it varies. On average, a good test-taker can solve a CR question within 1 minute and 15 seconds. For easy ones, 30-second is enough. For tough ones, 2-3 minutes are not uncommon.
One aspect of any standard test is to frustrate the test-taker and see if the frustration affects his/her performance. Another is to test his or her judgment to pick and choose. To spend 5 minutes on a single question is insane. Better work on a solvable question in stead of wasting time on a dead-end, futile attempt.
A question again
for Q1,my idea is: a jump exits between
ticketed for exceeding the speed limit and
more likely to exceed the speed limit ,while the (b) links the former concept with the latter one, as a result ,the (b) is more likely the corret answer than the rest.
is that right?
Besides, after reading your series, I can got correct answers more frequently, however , I have to spend a lot of time gaining them for I have to read and understand the stimulus and the options carefully. So what should I do for my pace now? How can I practise for both the high rate of correction and ideal pace? What is the ideal speed of one CR question for the test?
Thanks
-- by 会员 suntree846 (2011/10/3 19:30:48)
作者: suntree846 时间: 2011-10-3 22:31
受益良多哟~谢谢!
期待你的flaw(2)~
作者: QUARTZ7 时间: 2011-10-4 22:44
标题: Q2
Hi. dear SDCAR2010, for question2, I choose answer A, and still can't figure out what's wrong. Would you please explain it to me? Thank you very much!!!
作者: sdcar2010 时间: 2011-10-5 00:45
2. A researcher discovered that people who have low levels of immune-system activity tend to score much lower on tests of mental health than do people with normal or high immune-system activity. The researcher concluded from this experiment that the immune system protects against mental illness as well as against physical disease.
The researcher's conclusion depends on which of the following assumptions?
(A) High immune-system activity protects against mental illness better than normal immune system activity does.
(B) Mental illness is similar to physical disease in its effects on body systems.
(C) People with high immune-system activity cannot develop mental illness.
(D) Mental illness does not cause people's immune system activity to decrease.
(E) Psychological treatment of mental illness is not as effective as is medical treatment.
If you negate (D), you have:
Mental illness DOES cause people's immune system activity to decrease.
If that is true, then the author’s conclusion that the immune system protects against mental illness is wrong. So if you negate (D), the stimulus falls apart. Thus, (D) is the correct answer.
In A), there are two new concepts: high-immune system activity and normal-immune system activity. Both are NEW information, which are almost ALWAYS wrong for a NECESSARY assumption question simply because the new information is not required for the original passage/argument to hold.
Hi. dear SDCAR2010, for question2, I choose answer A, and still can't figure out what's wrong. Would you please explain it to me? Thank you very much!!!
-- by 会员 QUARTZ7 (2011/10/4 22:44:14)
作者: QUARTZ7 时间: 2011-10-5 15:39
2. A researcher discovered that people who have low levels of immune-system activity tend to score much lower on tests of mental health than do people with normal or high immune-system activity. The researcher concluded from this experiment that the immune system protects against mental illness as well as against physical disease.
The researcher's conclusion depends on which of the following assumptions?
(A) High immune-system activity protects against mental illness better than normal immune system activity does.
(B) Mental illness is similar to physical disease in its effects on body systems.
(C) People with high immune-system activity cannot develop mental illness.
(D) Mental illness does not cause people's immune system activity to decrease.
(E) Psychological treatment of mental illness is not as effective as is medical treatment.
If you negate (D), you have:
Mental illness DOES cause people's immune system activity to decrease.
If that is true, then the author’s conclusion that the immune system protects against mental illness is wrong. So if you negate (D), the stimulus falls apart. Thus, (D) is the correct answer.
In A), there are two new concepts: high-immune system activity and normal-immune system activity. Both are NEW information, which are almost ALWAYS wrong for a NECESSARY assumption question simply because the new information is not required for the original passage/argument to hold.
-- by 会员 sdcar2010 (2011/10/5 0:45:17)
Thanks!! But I'm still puzzled. Are high-immune system activity and normal immune system activity new information? The passage has mentioned them "people with normal or high immune-system activity " .
Or, can I say that A is wrong, because if I negate A: High immune system activity protects against mental illness worse than normal immune system activity does, it doesn't mean that immunne system can't protect against mental illness.
Hope for your explanation. Thank you very much!
作者: QUARTZ7 时间: 2011-10-5 16:20
Besides, dear SCARD2010, I don't understand clearly what's the importance of finding the conclusion and premises in the type of ASSUMPTION questions, as in the BF ones. Are the main uses finding the jumps and negating the answers? Thanks!!!
作者: sdcar2010 时间: 2011-10-5 20:36
Quart27, you are right about the high- normal-immune system NOT being new info. My bad.
However, both were treated as having the same effect when compared against low-immune system. The conclusion focused on the difference between low- and [high-, normal]. Since A focus on the difference between high- and normal-, it is not necessary for the the main conclusion to hold.
作者: QUARTZ7 时间: 2011-10-7 09:25
Quart27, you are right about the high- normal-immune system NOT being new info. My bad.
However, both were treated as having the same effect when compared against low-immune system. The conclusion focused on the difference between low- and [high-, normal]. Since A focus on the difference between high- and normal-, it is not necessary for the the main conclusion to hold.
-- by 会员 sdcar2010 (2011/10/5 20:36:08)
I'm really appreciated your help!
作者: bitterbeauty 时间: 2011-10-7 21:13
Hi, NN, that is very kind of u.
I have read the post, but i do not know why do we seperate sufficient assumptions from necessary assumptions?
I hardly distinguish them!!
作者: sdcar2010 时间: 2011-10-7 22:19
Sufficient assumption: with it, the conclusion will work; without it, the conclusion might or might not work.
Necessary assumption: without it, the conclusion will NOT work; with it, the conclusion might or might not work.
Statement: If this cat is yours, then it is a white cat.
Sufficient assumption: All of your cats are white.
Necessary assumption: At least one of your cats is white.
Hi, NN, that is very kind of u.
I have read the post, but i do not know why do we seperate sufficient assumptions from necessary assumptions?
I hardly distinguish them!!
-- by 会员 bitterbeauty (2011/10/7 21:13:14)
作者: bitterbeauty 时间: 2011-10-7 22:20
牛牛,sufficient和necessary分不清啊,牛牛给的例子看得很清楚,不过出在题目里面就分不清啦,感觉justify the conclusion 和 rely on the assumption差不多啊,求解答~
-- by 会员 dualtunnal (2011/7/15 22:15:12)
justify the conclusion: sufficient assumption
rely on the assumption: necessary assumption
-- by 会员 sdcar2010 (2011/7/16 0:03:39)
原来这里有...哈哈,明白了
作者: freyang 时间: 2011-10-9 16:30
Try this question:
The U.S. census is not perfect: thousands of Americans probably go uncounted. However, the basic statistical portrait of the nation painted by the census is accurate. Certainly some of the poor go uncounted, particularly the homeless; but some of the rich go uncounted as well, because they are often abroad or traveling between one residence and another.
Which of the following is an assumption on which the argument above depends?
(A) Both the rich and the poor have personal and economic reasons to avoid being counted by the census.
(B) All Americans may reasonably be classified as either poor or rich.
(C) The percentage of poor Americans uncounted by the census is close to the percentage of rich Americans uncounted.
(D) The number of homeless Americans is approximately equal to the number of rich Americans.
(E) The primary purpose of the census is to analyze the economic status of the American population.
-- by 会员 sdcar2010 (2011/6/28 20:58:35)
Dear sdcar201,
I am confused about the answer B.
According to my reasoning, if B is negated, that is to say, some Americans are classified to a third class, then the ratio of the poor to the rich to the third class can not be accurate even though some of the poor and of the rich are both uncounted.
Would you please give me a hand
作者: sdcar2010 时间: 2011-10-9 20:41
Necessary assumption. Use negation.
C is the right answer. If you negate C, then the basic statistical portrait of the nation painted by the census is NOT accurate since the missing poor and missing rich would distort the data.
B) when negated, can still make the conclusion work if what C) says is correct.
Basically, C) says if the mistaken percentage is the same for the rich and the poor, the census gives you a correct picture of the population.
Try this question:
The U.S. census is not perfect: thousands of Americans probably go uncounted. However, the basic statistical portrait of the nation painted by the census is accurate. Certainly some of the poor go uncounted, particularly the homeless; but some of the rich go uncounted as well, because they are often abroad or traveling between one residence and another.
Which of the following is an assumption on which the argument above depends?
(A) Both the rich and the poor have personal and economic reasons to avoid being counted by the census.
(B) All Americans may reasonably be classified as either poor or rich.
(C) The percentage of poor Americans uncounted by the census is close to the percentage of rich Americans uncounted.
(D) The number of homeless Americans is approximately equal to the number of rich Americans.
(E) The primary purpose of the census is to analyze the economic status of the American population.
-- by 会员 sdcar2010 (2011/6/28 20:58:35)
Dear sdcar201,
I am confused about the answer B.
According to my reasoning, if B is negated, that is to say, some Americans are classified to a third class, then the ratio of the poor to the rich to the third class can not be accurate even though some of the poor and of the rich are both uncounted.
Would you please give me a hand
-- by 会员 freyang (2011/10/9 16:30:49)
作者: chaseshi 时间: 2011-10-25 23:03
8. When people evade income taxes by not declaring taxable income, a vicious cycle results. Tax evasion forces lawmakers to raise income tax rates, which causes the tax burden on non-evading taxpayers to become heavier. This, in turn, encourages even more taxpayers to evade income taxes by hiding taxable income.
The vicious cycle described above could not result unless which of the following were true?
(D) No one who routinely hides some taxable income can be induced by a lowering of tax rates to stop hiding such income unless fines for evaders are raised at the same time.
-------------------------------------------------------------------------------------------------------------------------
LZ, just wondering is "Tax evasion forces lawmakers to raise income tax rates, which causes the tax burden on non-evading taxpayers to become heavier." the conclusion of the argument? And is the last sentence just the inference?
Secondly, why is selection D wrong? Does the sentence "...by a lowering of tax rates" try to contradict the conclusion?
作者: bonfin 时间: 2011-10-31 10:23
up
作者: blueday5 时间: 2011-10-31 20:11
Hi, NN, 我看了好一段时间才把Sufficient assumption和Necessary assumption想通
但是充分假设和必要假设要怎么实际应用到做题呢?
例如,
2. A researcher discovered that people who have low levels of immune-system activity tend to score much lower on tests of mental health than do people with normal or high immune-system activity. The researcher concluded from this experiment that the immune system protects against mental illness as well as against physical disease.
The researcher's conclusion depends on which of the following assumptions?
(A) High immune-system activity protects against mental illness better than normal immune system activity does.
(B) Mental illness is similar to physical disease in its effects on body systems.
(C) People with high immune-system activity cannot develop mental illness.
(D) Mental illness does not cause people's immune system activity to decrease.
(E) Psychological treatment of mental illness is not as effective as is medical treatment.
通过题目说:The researcher's conclusion depends on which of the following assumptions?
我知道这题是在问必要假设
想请问接下来看选项时,Sufficient Assumption or Necessary Assumption 还有其他功用吗?
我现在的困惑是,看了LZ在#13的解释, 既然每道题都直接拿来negation,能weaken到原题即答案的话,为什么还需要去区别做题时看到的Assumption属于Sufficient或Necessary呢?(如果这样, 我仔细研究如何分辨Sufficient还是Necessary不就白忙一场了吗?)
不懂啊~~~,请求NN 再说明一下
作者: 无敌西红柿炒蛋 时间: 2011-11-1 17:23
Hi,Sdcar!
For Q 1,the premise is "33 percent of all vehicles ticketed for exceeding the speed limit were equipped with them". And the main conclusion is "drivers who equip their vehicles with radar detectors are more likely to exceed the speed limit regularly than are drivers who do not."
If my judgement is correct, and what's the gap between the premise and the conclusion. In view,through the condition and the premise ,we can logically deduce the conclusion.
Could you explain it explicitly ?This question confuse me a lot.
THX
作者: sdcar2010 时间: 2011-11-2 02:19
Hi,Sdcar!
For Q 1,the premise is "33 percent of all vehicles ticketed for exceeding the speed limit were equipped with them". And the main conclusion is "drivers who equip their vehicles with radar detectors are more likely to exceed the speed limit regularly than are drivers who do not."
If my judgement is correct, and what's the gap between the premise and the conclusion. In view,through the condition and the premise ,we can logically deduce the conclusion.
Could you explain it explicitly ?This question confuse me a lot.
THX
-- by 会员 无敌西红柿炒蛋 (2011/11/1 17:23:49)
What if the policeman hates those drivers who install radar detector and gives them speeding tickets whenever the policeman sees any of them???
作者: sdcar2010 时间: 2011-11-2 02:21
Hi, NN, 我看了好一段时间才把Sufficient assumption和Necessary assumption想通
但是充分假设和必要假设要怎么实际应用到做题呢?
例如,
2. A researcher discovered that people who have low levels of immune-system activity tend to score much lower on tests of mental health than do people with normal or high immune-system activity. The researcher concluded from this experiment that the immune system protects against mental illness as well as against physical disease.
The researcher's conclusion depends on which of the following assumptions?
(A) High immune-system activity protects against mental illness better than normal immune system activity does.
(B) Mental illness is similar to physical disease in its effects on body systems.
(C) People with high immune-system activity cannot develop mental illness.
(D) Mental illness does not cause people's immune system activity to decrease.
(E) Psychological treatment of mental illness is not as effective as is medical treatment.
通过题目说:The researcher's conclusion depends on which of the following assumptions?
我知道这题是在问必要假设
想请问接下来看选项时,Sufficient Assumption or Necessary Assumption 还有其他功用吗?
我现在的困惑是,看了LZ在#13的解释, 既然每道题都直接拿来negation,能weaken到原题即答案的话,为什么还需要去区别做题时看到的Assumption属于Sufficient或Necessary呢?(如果这样
, 我仔细研究如何分辨Sufficient还是Necessary不就白忙一场了吗?)
不懂啊~~~,请求NN 再说明一下
-- by 会员 blueday5 (2011/10/31 20:11:04)
Since all those problems I posted are NECESSARY assumption questions. Therefore, negation is used for all of them. If you see a sufficient assumption question, DO NOT use negation method.
Try this one:
The average thirty-year-old Chinese will have watched more than ten thousand TV commercials in his or her lifetime. Therefore, the TV commercial has influenced Chinese way of thinking.
The conclusion above follows logically if which one of the following is assumed?
A) The way of thinking which people develop are largely determined by external influences.
B) Anything people are exposed to in great quantity will influence their way of thinking.
C) It is impossible to avoid or ignore TV commercials.
D) Some peole find TV commercials more interesting to watch than the news broadcast every night.
E) Certain form of communication to which people are subjected will affect their way of thinking.
欢迎光临 ChaseDream (https://forum.chasedream.com/) |
Powered by Discuz! X3.3 |